Đến nội dung

rfiyms nội dung

Có 20 mục bởi rfiyms (Tìm giới hạn từ 29-04-2020)


Sắp theo                Sắp xếp  

#653444 $\sum ab.(\sum\sqrt{ab} )^2\geq 27$

Đã gửi bởi rfiyms on 08-09-2016 - 23:54 trong Bất đẳng thức và cực trị

Cho a,b,c thực dương thỏa : $\sum a= \sum \frac{1}{a}$

CM: $\sum ab.(\sum\sqrt{ab} )^2\geq 27$

Đặt $x=ab$, $y=bc$, $z=ca$ thì $a=\sqrt{\dfrac{zx}{y}}$, $b=\sqrt{\dfrac{xy}{z}}$, $c=\sqrt{\dfrac{yz}{x}}$ và $x+y+z=xy+yz+zx$.

Bất đẳng thức cần chứng minh trở thành:

$$\left ( x+y+z \right )\left ( \sqrt{xy}+\sqrt{yz}+\sqrt{zx} \right )^{2}\geq 27$$

$$\Leftrightarrow \sqrt{x}+\sqrt{y}+\sqrt{z}\geq \dfrac{3\sqrt{3}}{\sqrt{x+y+z}}$$
Viết lại bất đẳng thức dưới dạng:
$$\sqrt{\dfrac{x}{x+y+z}}+\sqrt{\dfrac{y}{x+y+z}}+\sqrt{\dfrac{z}{x+y+z}}\geq \dfrac{3\sqrt{3}\left ( xy+yz+zx \right )}{\left ( x+y+z \right )^{2}}$$
$$\Leftrightarrow 2\left ( \sqrt{\dfrac{x}{x+y+z}}+\sqrt{\dfrac{y}{x+y+z}}+\sqrt{\dfrac{z}{x+y+z}} \right )+\dfrac{3\sqrt{3}\left ( xy+yz+zx \right )}{\left ( x+y+z \right )^{2}}\geq 3\sqrt{3}$$
Theo bất đẳng thức AM-GM ta có:
$$\dfrac{3\sqrt{3}x^{2}}{\left ( x+y+z \right )^{2}}+\sqrt{\dfrac{x}{x+y+z}}+\sqrt{\dfrac{x}{x+y+z}}\geq \dfrac{3\sqrt{3}x}{x+y+z}$$
Chứng minh tương tự rồi cộng lại ta được bất đẳng thức cần chứng minh.
Đẳng thức xảy ra khi $x=y=z=1$.



#653137 CMR: pq + 6 $\geq$ 5p

Đã gửi bởi rfiyms on 07-09-2016 - 17:37 trong Bất đẳng thức và cực trị

Cho a,b,c>0, p=a+b+c, q=ab+bc+ca, r=abc=1

CMR: pq + 6 $\geq$ 5p  

Hình như bạn ghi nhầm đề, đề phải là:

$$\left ( a+b+c \right )\left ( ab+bc+ca \right )+6\geq 5\left ( ab+bc+ca \right )$$

Nếu đề đúng như mình ghi thì đây là lời giải của thầy Đặng Thành Nam:

Bất đẳng thức trên tương đương:

$$a+b+c+\dfrac{6}{ab+bc+ca}\geq 5$$

Đặt $P\left ( a,b,c \right )=a+b+c+\dfrac{6}{ab+bc+ca}-5$.

Giả sử $a=\min \left \{ a;b;c \right \}$ thì $a\leq 1$. Do đó:

$$P\left ( a,b,c \right )-P\left ( a,\sqrt{bc},\sqrt{ca} \right )=b+c-2\sqrt{bc}+\dfrac{6}{a\left ( b+c \right )+bc}-\dfrac{6}{2a\sqrt{bc}+bc}=\left ( \sqrt{b}-\sqrt{c} \right )^{2}\left [ 1-\dfrac{6a}{\left ( ab+bc+ca \right )\left ( bc+2a\sqrt{bc} \right )} \right ]\geq \left ( \sqrt{b}-\sqrt{c} \right )^{2}\left ( 1-\dfrac{2a}{3} \right )\geq 0$$

Do đó ta cần chứng minh:

$$P\left ( \dfrac{1}{t^{2}},t,t \right )\geq 0, \ \forall t\geq 1$$

$$\Leftrightarrow \dfrac{1}{t^{2}}+2t+\dfrac{6}{\frac{1}{t^{2}}+t^{2}}-5\geq 0$$
$$\Leftrightarrow \left ( t-1 \right )\left ( 2t^{5}-3t^{4}-3t^{3}+8t^{2}-2 \right )\geq 0, \ \forall t\geq 1$$
Bất đẳng thức cuối đúng nên bài toán được chứng minh.
Cách khác ngắn gọn hơn:
Ta có:
$$a^{2}+b^{2}+c^{2}+2abc+1\geq 2\left ( ab+bc+ca \right )$$
$$\Leftrightarrow a^{2}+b^{2}+c^{2}+3\geq 2\left ( ab+bc+ca \right )$$
$$\Leftrightarrow \left ( a+b+c \right )^{2}+3\geq 4\left ( ab+bc+ca \right )$$
Do đó ta cần chứng minh:
$$a+b+c+\dfrac{24}{\left ( a+b+c \right )^{2}+3}\geq 5$$
$$\Leftrightarrow \left ( a+b+c \right )^{3}-5\left ( a+b+c \right )^{2}+3\left ( a+b+c \right )+9\geq 0$$
$$\Leftrightarrow \left ( a+b+c+1 \right )\left ( a+b+c-3 \right )^{2}\geq 0$$



#653006 Chứng minh rằng : $\frac{1}{2-a}$ + $...

Đã gửi bởi rfiyms on 06-09-2016 - 15:25 trong Bất đẳng thức và cực trị

Cho 3 số dương a,b,c thỏa mãn $a^{2}$ + $b^{2}$ + $c^{2}$ =3

Chứng minh rằng :$\frac{1}{2-a}$ + $\frac{1}{2-b}$ + $\frac{1}{2-c}$  $\geq$ 3

Cách khác dùng phương pháp mình rất thích: $p$, $q$, $r$.

Quy đồng và thu gọn ta được bất đẳng thức cần chứng minh trở thành:

$$8p+3r\geq 12+5q$$

Áp dụng bất đẳng thức Schur ta có:

$$3r\geq \dfrac{p\left ( 4q-p^{2} \right )}{3}=\dfrac{p\left ( 2q-3 \right )}{3}$$

$$\Leftrightarrow 8p+\dfrac{p\left ( p^{2}-6 \right )}{3}\geq 12+\dfrac{5\left ( p^{2}-3 \right )}{2}$$

$$\Leftrightarrow \left ( 2p-3 \right )\left ( p-3 \right )^{2}\geq 0$$

Bất đẳng thức cuối đúng, đẳng thức xảy ra khi $a=b=c=1$.




#652866 $a+b+c+3abc\geq 6$

Đã gửi bởi rfiyms on 05-09-2016 - 11:06 trong Bất đẳng thức và cực trị

cho các số không âm thỏa mạn $ab+bc+ac+6abc=9$.Chứng minh rằng: 

$a+b+c+3abc\geq 6$

Đề bài có chứa $a+b+c$, $ab+bc+ca$ và $abc$ nên một cách tự nhiên ta nghĩ đến phương pháp $p$, $q$, $r$.

Đặt $a+b+c=p$, $ab+bc+ca=q$, $abc=r$ khi đó ta có $p\geq 3$.

Với $p\geq 6$ thì bất đẳng thức hiển nhiên đúng nên ta chỉ xét $3\leq p\leq 6$.

Theo bất đẳng thức Schur ta có $p^{3}+9r\geq 4pq\Leftrightarrow 2p^{3}+3\left ( 9-q \right )\geq 8pq\Rightarrow q\leq \dfrac{2p^{3}+27}{8p+3}$.

Vậy ta cần chứng minh $p+3r\geq 6\Leftrightarrow 2p+9-q\geq 12\Leftrightarrow 2p-q-3\geq 0$.

Mặt khác ta có bất đẳng thức mạnh hơn là $2p-\dfrac{2p^{3}+27}{8p+3}-3\geq 0$ bất đẳng thức này chứng minh bằng biến đổi tương đương ta được $\left ( 6-p \right )\left ( p-3 \right )\left ( p+1 \right )\geq 0$ (đúng vì $3\leq p\leq 6$).

Đẳng thức xảy ra khi $a=b=c=1$ hoặc $a=b=3$, $c=0$ và các hoán vị.




#652657 Hỏi có bao nhiêu cách bỏ tất cả các quả cầu vào trong các hộp

Đã gửi bởi rfiyms on 03-09-2016 - 21:44 trong Tổ hợp - Xác suất và thống kê - Số phức

Cho $m$ quả cầu đôi một khác nhau và $n$ cái hộp đôi một khác nhau ($m\geq n$). Hỏi có bao nhiêu cách bỏ tất cả các quả cầu vào trong các hộp sao cho hộp nào cũng có ít nhất $1$ qủa cầu (không kể thứ tự các quả cầu trong mỗi hộp).




#652646 CM : $2a+7 \vdots 7\Leftrightarrow 3a^2 +10ab-8b^2 \vdots...

Đã gửi bởi rfiyms on 03-09-2016 - 20:51 trong Số học

Bài 2 : Cho p là số nguyên tố lớn hơn 5 . Chứng minh rằng nếu n là số tự nhiên có p-1 chữ số và các chữ số đó đều bằng 1 thì n chia hết p

Gọi $n=111...1=n= \underbrace{11...1}_{p-1}= \frac{10^{p-1}-1}{9}$.

Vì $\gcd (p,10)=1$ nên theo định lý Fermat nhỏ $p\mid 10^{p-1}-1$.

Mặt khác vì $p$ là số nguyên tố lớn hơn $5$ nên $\gcd (p,3)=1$.

Suy ra $p\mid \frac{10^{p-1}-1}{9}$ hay $p\mid n$.




#652008 Tìm $a,b,p$ thỏa mãn: $2^a+p^b=19^a.$

Đã gửi bởi rfiyms on 30-08-2016 - 21:08 trong Số học



Bài toán $(Italy \;\;\ TST  \;\;\ 2003):$ Tìm bộ số nguyên $(a,b,p)$ sao cho $a,b$ là số nguyên dương và $p$ là số nguyên tố thỏa mãn:

$$2^a+p^b=19^a.$$ 

Lời giải của frill trên AoPS:

Another way to solve it is to use the Lifting the exponent lemma (LTE).

So noting $p=17$ we need to solve $2^{a}+17^{b}=19^{a}=(2+17)^{a}>2^{a}+17^{a}\ \forall a>1$
$\therefore b\geq a+1$ or $a=1$ if the latter then $a=b=1$ otherwise:
$17\mid19-2$ and $17\nmid19,17\nmid2$,
$\therefore$ by LTE: $v_{17}(19^{a}-2^{a})=v_{17}(19-2)+v_{17}(a)$
$\Leftrightarrow b=1+v_{17}(a)$
$\therefore 1+v_{17}(a)\geq a+1$
$\Leftrightarrow v_{17}(a)\geq a$ which is untrue $\forall a \in Z>0$
So the only solution for $(a,b,p)$ is $(1,1,17)$
 
Lời giải của WakeUp trên AoPS:

Anyone can see $p=17$.

Looking modulo $9$, we can establish $a=1\pmod{6}$. Note $a=1\iff b=1$, the solution $(1,1)$ is produced, otherwise $a\ge 7$.
Now let's rearrange: $2^a+17^b=19^a$ which means that $17^b=(19-2)(19^{a-1}+2\cdot 19^{a-2}+2^2\cdot 19^{a-3}\ldots + 2^{a-1})$.
Hence $17^{b-1}=(19^{a-1}+2\cdot 19^{a-2}+2^2\cdot 19^{a-3}\ldots + 2^{a-1})$. 
The latter equation writes as $1=3^{a-1}+2\cdot 3^{a-2} \pmod{4}$. Since $a=1\pmod{6}$, the last equation cannot hold since $a$ is odd ($LHS=1$ while $RHS=3$). So only solution is $(1,1,17)$.
 

Lời giải khác rất hay của Vax trên AoPS:

$p^b = 19^a-2^a$

If $a > 1$ then with Zsigmondy's Theorem there exists a prime p, such that $p \mid 19^a-2^a \wedge p \nmid 19-2 = 17$, so $a=1 \rightarrow p^b = 17 \rightarrow b=1 \wedge p=17$

 




#651487 Cho số thực a thuộc (0;1), xet day $(u_n)$ voi

Đã gửi bởi rfiyms on 27-08-2016 - 15:39 trong Dãy số - Giới hạn

Cho số thực a thuộc (0;1), xet day $(u_n)$ voi.

$\left\{\begin{array}{1}u_1 =a \\ u_{n+1} \frac{1}{2017} u^2_n +\frac{2016}{2017}\sqrt{u_n}, n thuoc N^*  \end{array}\right.$

a) chứng minh rằng: $0<u_n<1$, mọi n thuộc $N^*$.

b) Chứng minh $(u_n)$ có giới hạn hữu hạn. Tìm giới hạn đó.

a) Ta có $u_{1}=a\in \left ( 0;1 \right )$.

Giả sử $u_{n}\in \left ( 0;1 \right )$ ta sẽ chứng minh $u_{n+1}\in \left ( 0;1 \right )$.

Thật vậy ta có:

$$0<u_{n+1}=\dfrac{1}{2017}u_{n}^{2}+\dfrac{2016}{2017}\sqrt{u_{n}}<\dfrac{1}{2017}.1^{2}+\dfrac{2016}{2017}.\sqrt{1}=1$$

Vậy ta luôn có $0<u_{n}<1$ với mọi $n\in \mathbb{N^{*}}$.

b) Ta có:

\begin{align*} u_{n+1}-u_{n} &=\dfrac{1}{2017}u_{n}^{2}-u_{n}+\dfrac{2016}{2017}\sqrt{u_{n}} \\ &=\dfrac{x}{2017}\left ( x^{3}-2017x+2016 \right ) \\ &=\dfrac{x\left ( x-1 \right )\left ( x^{2}+x-2016 \right )}{2017}>0, \,\,\,\, \forall x=u_{n}\in \left ( 0;1 \right ) \end{align*}

Do đó $\left ( u_{n} \right )$ là dãy tăng và bị chặn trên bởi $1$ nên dãy có giới hạn hữu hạn.

Ta có:

$$\lim _{n\rightarrow +\infty }u_{n}=\lim _{n\rightarrow +\infty }u_{n}\left ( \dfrac{1}{2017}u_{n}^{2}+\dfrac{2016}{2017}\sqrt{u_{n}} \right )$$

Đặt $\lim _{n\rightarrow +\infty }u_{n}=x\in \left ( 0;1 \right )$ thì ta có:

$$x=\dfrac{1}{2017}x^{2}+\dfrac{2016}{2017}\sqrt{x}$$

$$\Leftrightarrow x=1$$
Vậy $\lim _{n\rightarrow +\infty }u_{n}=1$.



#651486 Cho x, y, z là các số thực dương thỏa mãn điều kiện $x^3+y^2+z=2\sq...

Đã gửi bởi rfiyms on 27-08-2016 - 15:29 trong Bất đẳng thức và cực trị

Cho x, y, z là các số thực dương thỏa mãn điều kiện $x^3+y^2+z=2\sqrt{3}+1$. Tìm giá trị nhỏ nhất của biểu thức

$P=\frac{1}{x}+\frac{1}{y^2}+\frac{1}{z^3}$

Ta sẽ tìm số thực dương $k$ sao cho sử dụng được giả thuyết $x^{3}+y^{2}+z=2\sqrt{3}+1$ như sau:

$$P+k\left ( x^{3}+y^{3}+z \right )\geq m$$

($m$ là giá trị nhỏ nhất cần tìm).

Theo bất đẳng thức AM-GM ta có:

$$\dfrac{1}{x}+kx^{3}=\dfrac{1}{3x}+\dfrac{1}{3x}+\dfrac{1}{3x}+kx^{3}\geq 4\sqrt[4]{\dfrac{k}{27}}$$

$$\dfrac{1}{y^{2}}+ky^{2}\geq 2\sqrt{k}$$
$$\dfrac{1}{z^{3}}+kz=\dfrac{1}{z^{3}}+\dfrac{kz}{3}+\dfrac{kz}{3}+\dfrac{kz}{3}\geq 4\sqrt[4]{\dfrac{k^{3}}{27}}$$
$$\Rightarrow P+k\left ( x^{3}+y^{3}+z \right )\geq 4\sqrt[4]{\dfrac{k}{27}}+2\sqrt{k}+4\sqrt[4]{\dfrac{k^{3}}{27}}$$
Dấu bằng xảy ra khi và chỉ khi:
$$\left\{\begin{matrix} \dfrac{1}{3x}=kx^{3} \\ \dfrac{1}{y^{2}}=ky^{2} \\ \dfrac{1}{z^{3}}=\dfrac{kz}{3} \\ x^{3}+y^{2}+z=2\sqrt{3}+1 \end{matrix}\right.$$
$$\Leftrightarrow \left ( x;y;z;k \right )=\left ( 1;\sqrt[4]{3};\sqrt{3};\dfrac{1}{3} \right )$$
Vậy giá trị nhỏ nhất của $P$ là $\dfrac{9+4\sqrt{3}}{9}$ đạt được khi $\left ( x;y;z \right )=\left ( 1;\sqrt[4]{3};\sqrt{3} \right )$.



#651484 Tìm tất cả các cặp số nguyên (x, y) thỏa mãn: $x(1+x+x^2)=4y(y-1)$.

Đã gửi bởi rfiyms on 27-08-2016 - 15:14 trong Số học

b)  Trên mặt phẳng cho 2017 điểm sao cho với ba điểm bất kỳ trong số các điểm đó ta luôn tìm được 2 điểm để đoạn thẳng tạo thành có độ dài bé hơn 1. Chứng minh luôn tồn tại một hình tròn bán kính bằng 1 chứa không ít hơn 1009 điểm đã cho.

Gọi $A_{1},A_{2},..,A_{2017}$ là các điểm đã cho trên mặt phẳng.

Nếu tất cả các đoạn thẳng được tạo ra từ $2$ điểm trong $2017$ điểm đã cho có độ dài bé hơn $1$. Như vậy các điểm đã cho nằm trong đường tròn tâm $A_{1}$ bán kính bằng $1$.
Nếu tồn tại đoạn thẳng có độ dài lớn hơn hoặc bằng $1$. Giả sử $A_{1}A_{2}$ là đoạn thẳng có độ dài lớn nhất. Khi đó ta xét $3$ điểm $A_{1}$, $A_{2}$, $A_{k}$ với $k=\overline{3;2017}$.
Do $A_{1}A_{2}$ có độ dài lớn hơn hoặc bằng $1$ nên một trong hai đoạn thẳng $A_{1}A_{k}$ và $A_{2}A_{k}$ có độ dài nhỏ hơn $1$.
Gọi $B_{1}$ là số đoạn thẳng $A_{1}A_{k}$ có độ dài nhỏ hơn $1$, $B_{2}$ là số đoạn thẳng $A_{2}A_{k}$ có độ dài nhỏ hơn $1$.

Theo nguyên lý Dirichlet tồn tại một trong hai số $B_{1}$, $B_{2}$ lớn hơn hoặc bằng $1008$, giả sử là $B_{1}$.

Như vậy tồn tại ít nhất $1009$ điểm trong đường tròn tâm $A_{1}$ bán kính bằng $1$ kể cả điểm $A_{1}$.
Tóm lại ta luôn tìm được một đường tròn thỏa mãn yêu cầu bài toán.



#650823 CMR:$\sum \frac{a^{2}}{b^{2...

Đã gửi bởi rfiyms on 22-08-2016 - 18:52 trong Bất đẳng thức và cực trị

Cho a,b,c >0 CMR:$\sum \frac{a^{2}}{b^{2}+c^{2}}\geq \sum \frac{a}{b+c}$

Cách khác không sử dụng $\text{S.O.S}$:
$$\sum \frac{a^{2}}{b^{2}+c^{2}}\geq \sum \frac{a}{b+c}$$
$$\Leftrightarrow \sum \left ( a^{2}+\sum ab \right )\left [ \sum \frac{ab\left ( b-c \right )^{2}}{\left ( b+c \right )\left ( c+a \right )\left ( b^{2}+c^{2} \right )\left ( c^{2}+a^{2} \right )} \right ]\geq 0$$
Bài toán tổng quát có trong Sáng tạo bất đẳng thức của tác giả Phạm Kim Hùng.



#650822 CMR:$\sum \frac{a^{2}}{b^{2...

Đã gửi bởi rfiyms on 22-08-2016 - 18:51 trong Bất đẳng thức và cực trị

Cho a,b,c >0 CMR:$\sum \frac{a^{2}}{b^{2}+c^{2}}\geq \sum \frac{a}{b+c}$

Ta có:
\begin{align*} \text{VT}-\frac{3}{2} &=\frac{1}{2}\sum \left ( \frac{a^{2}}{b^{2}+c^{2}}-1 \right ) \\ &=\frac{1}{2}\sum \frac{\left ( a-b \right )\left ( a+b \right )+\left ( a-c \right )\left ( a+c \right )}{b^{2}+c^{2}} \\ &=\frac{1}{2}\sum \left [ \frac{\left ( a-b \right )\left ( a+b \right )}{b^{2}+c^{2}}-\frac{\left ( a-b \right )\left ( a+b \right )}{a^{2}+c^{2}} \right ] \\ &=\frac{1}{2}\left ( a-b \right )\left ( a+b \right )\frac{\left ( a-b \right )\left ( a+b \right )}{\left ( b^{2}+c^{2} \right )\left ( a^{2}+c^{2} \right )} \\ &=\frac{1}{2}\sum \frac{\left ( a-b \right )^{2}\left ( a+b \right )^{2}}{\left ( b^{2}+c^{2} \right )\left ( a^{2}+c^{2} \right )} \end{align*}
Tương tự:
\begin{align*} \text{VP}-\frac{3}{2} &=\frac{1}{2}\sum \frac{a-b+a-c}{b+c} \\ &=\frac{1}{2}\sum \left ( \frac{a-b}{b+c}-\frac{a-b}{a+c} \right ) \\ &=\frac{1}{2}\sum \frac{\left ( a-b \right )^{2}}{\left ( b+c \right )\left ( a+c \right )} \\ \end{align*}
Trừ vế theo vế hai bất đẳng thức trên ta được:
$$\text{VT}-\text{VP}=\frac{1}{2}\sum \left ( a-b \right )^{2}\left [ \frac{\left ( a+b \right )^{2}}{\left ( b^{2}+c^{2} \right )\left ( a^{2}+c^{2} \right )}-\frac{1}{\left ( b+c \right )\left ( a+c \right )} \right ]$$
Theo tiêu chuẩn $2$ của phương pháp $\text{S.O.S}$ bất đẳng thức cuối đúng do đó ta thu được bất đẳng thức cần chứng minh.



#650464 ĐỀ THI HỌC SINH GIỎI LỚP 12 THPT CẤP THÀNH PHỐ (HỒ CHÍ MINH)

Đã gửi bởi rfiyms on 20-08-2016 - 10:24 trong Thi HSG cấp Tỉnh, Thành phố. Olympic 30-4. Đề thi và kiểm tra đội tuyển các cấp.

 Bài 6: Trong một buổi tọa đàm về "Tình yêu tuổi học đường" tại lớp $12A$, có tất cả $21$ bạn tham gia và có $4$ cặp có tình cảm với nhau( không có học sinh nào thuộc về nhiều cặp). Cô giáo chọn ra $5$ bạn để tham gia một trò chơi tập thể. Hỏi có bao nhiêu cách chọn mà trong đó, có ít nhất $1$ cặp tình cảm với nhau

Gọi $A$ là nhóm gồm các học sinh có tình cảm với nhau (gồm có $8$ học sinh) và $B$ là nhóm các học sinh còn lại (có $13$ học sinh).

$\bullet $ Trường hợp $1$: Có đúng $1$ cặp có tình cảm với nhau.

Đầu tiên chọn $1$ cặp có tình cảm với nhau thì có $4$ cách chọn.

Tiếp theo chọn $3$ học sinh trong đó không có $2$ em nào có tình cảm với nhau thì có những trường hợp sau:

- $3$ học sinh này thuộc nhóm $A$ thì có $2^{3}$ cách chọn.

- $2$ học sinh thuộc nhóm $A$, $1$ học sinh thuộc nhóm $B$ thì có $C_{3}^{2}.2^{2}.C_{13}^{1}$ cách chọn.

- $1$ học sinh thuộc nhóm $A$, $2$ học sinh thuộc nhóm $B$ thì có $C_{3}^{1}.2.C_{13}^{2}$ cách chọn.

- $3$ học sinh thuộc nhóm $B$ thì có $C_{13}^{3}$ cách chọn.

$\bullet $ Trường hợp $1$: Có đúng $2$ cặp có tình cảm với nhau.

Đầu tiên chọn $2$ cặp có tình cảm với nhau có $C_{4}^{2}$ cách chọn.
Học sinh còn lại có $C_{17}^{1}$ cách chọn.
Vậy tổng cộng có $4\left ( 2^{3}+C_{3}^{2}.2^{2}.C_{13}^{1}+C_{3}^{1}.2.C_{13}^{2}+C_{13}^{3} \right )+6.C_{17}^{1}=3774$ cách chọn.
Bài dùng từ kì quá, thay "có tình cảm" bằng "biết nhau" hoặc từ khác hay hơn :D



#650460 ĐỀ THI HỌC SINH GIỎI LỚP 12 THPT CẤP THÀNH PHỐ (HỒ CHÍ MINH)

Đã gửi bởi rfiyms on 20-08-2016 - 10:02 trong Thi HSG cấp Tỉnh, Thành phố. Olympic 30-4. Đề thi và kiểm tra đội tuyển các cấp.

Bài 3: Cho $x,y,z$ là các số thực thỏa mãn: $x^2+y^2+\frac{3}{2}=2(x+y)$. Tìm GTLN và GTNN của biểu thức:

$P=\frac{6-2(x-1)(y-1)}{(x-1)^2+(y-1)^2}$

Giả thuyết tương đương:

$$\left ( x-1 \right )^{2}+\left ( y-1 \right )^{2}=\dfrac{1}{2}$$

Đặt $x-1=a$, $y-1=b$ thì $a^{2}+b^{2}=\dfrac{1}{2}$ và $P$ trở thành:

$$P=\dfrac{6-2ab}{a^{2}+b^{2}}=12-4ab$$

Mặt khác với mọi số thực $a$, $b$ ta có bất đẳng thức sau:

$$-\left ( a^{2}+b^{2} \right )\leq 2ab\leq a^{2}+b^{2}$$

Dấu bằng xảy ra lần lượt khi $a=-b$ và $a=b$.

Do đó ta có $11\leq P\leq 13$.

Vậy $P_{\min }=11$ đạt được khi $x=y=\dfrac{1}{2}$, $P_{\max }=13$ đạt được khi $x=\dfrac{3}{2}$, $y=\dfrac{1}{2}$.




#650226 Tìm số nguyên dương $k$ nhỏ nhất để không có hai số nào cùng màu là...

Đã gửi bởi rfiyms on 18-08-2016 - 15:18 trong Tổ hợp và rời rạc

Bài toán. Tìm số nguyên dương $k$ nhỏ nhất sao cho ta có thể tô màu các số nguyên dương từ $1$ đến $n$ bằng $k$ màu khác nhau để không có hai số nào cùng màu là bội của nhau.

Để ý rằng $2^{\left \lfloor \log _{2}n \right \rfloor}=\left \lfloor n \right \rfloor\leq n$.

Xét dãy các số $2^{0};2^{1};2^{2};...;2^{\left \lfloor \log _{2}n \right \rfloor}$ thì ta thấy không thể có hai số nào được tô cùng một màu.

Do đó $k\geq \left \lfloor \log _{2}n \right \rfloor+1$.

Bây giờ ta sẽ chứng minh với $k=\left \lfloor \log _{2}n \right \rfloor+1$ thì ta có thể thực hiện được cách tô màu theo yêu cầu bài toán.

Thật vậy với $k=\left \lfloor \log _{2}n \right \rfloor+1$ ta tô màu như sau:

Đặt $A_{i}=\left \{ 2^{i};2^{i}+1;2^{i}+2;...;2^{i+1}-1 \right \} \,\,\,\, \left ( i=\overline{0;\left \lfloor \log _{2}n \right \rfloor-1} \right )$ và $A_{j}=\left \{ 2^{\left \lfloor \log _{2}n \right \rfloor};2^{\left \lfloor \log _{2}n \right \rfloor}+1;2^{\left \lfloor \log _{2}n \right \rfloor}+2;...;n \right \}$.

Tô màu các số thuộc $A_{i}$ bởi màu thứ $i+1$ thì cách tô màu này thỏa mãn yêu cầu bài toán.

Vậy $k_{\min }=\left \lfloor \log _{2}n \right \rfloor+1$.




#650100 Giải phương trình bằng phương pháp đặt ẩn phụ.

Đã gửi bởi rfiyms on 17-08-2016 - 19:13 trong Phương trình - Hệ phương trình - Bất phương trình

Giải phương trình: 

a) $\frac{6x^{2}+17x+11}{2+\sqrt{(x+1)(6x+11)}}=\frac{27}{28}x+\frac{41}{14}.$

Điều kiện xác định: $x\geq -1$ hoặc $x\leq -\dfrac{11}{6}$.

Xét $x=-1$ không phải là nghiệm của phương trình do đó xét $x\neq -1$ và đặt:

$$\sqrt{\left ( x+1 \right )\left ( 6x+11 \right )}=\left ( x+1 \right )t\geq 0$$

$$\Rightarrow \left ( x+1 \right )\left ( 6x+11 \right )=\left ( x+1 \right )^{2}t^{2}$$
$$\Leftrightarrow \left ( t^{2}-6 \right )x=11-t^{2}$$
Dễ thấy $t=\pm \sqrt{6}$ thì không thỏa mãn phương trình do đó $t\neq \pm \sqrt{6}$ suy ra $x=\dfrac{-t^{2}+11}{t^{2}-6}$ thay vào phương trình ban đầu ta được:
$$\dfrac{\left ( \frac{5t}{t^{2}-6} \right )^{2}}{2+\frac{5t}{t^{2}-6}}=\dfrac{27\left ( 11-t^{2} \right )}{28\left ( t^{2}-6 \right )}+\dfrac{41}{14}$$
$$\Leftrightarrow \left ( t-1 \right )\left ( t-3 \right )\left ( 22t^{2}+143t+156 \right )=0$$
Đối chiếu với điều kiện thì ta được $t=3$ và $t=\dfrac{-143+\sqrt{6721}}{44}$ từ đó tìm được $x=\dfrac{2}{3}$ và $x=\dfrac{286\sqrt{6721}-5874}{15554-286\sqrt{6721}}$.
 

Giải phương trình: 

b) $(1+\sqrt{\frac{7}{3}x^{2}+\frac{2}{3}x+1})^{3}=\frac{11x^{3}+70x^{2}}{3}.$

Ta thấy $x=0$ không phải là nghiệm của phương trình do đó xét $x\neq 0$ và đặt:

$$\sqrt{\dfrac{7}{3}x^{2}+\dfrac{2}{3}x+1}=xt-1\geq 0$$

$$\Rightarrow \dfrac{7}{3}x^{2}+\dfrac{2}{3}x+1=\left ( xt-1 \right )^{2}$$

$$\Leftrightarrow \left ( 3t^{2}-7 \right )x=6t+2$$
Dễ thấy khi $t=\pm \sqrt{\dfrac{7}{3}}$ thì không thỏa mãn phương trình do đó $t\neq \pm \sqrt{\dfrac{7}{3}}$ ta được $x=\dfrac{6t+2}{3t^{2}-7}$ thay vào phương trình ban đầu ta được:
$$\left ( 1+xt-1 \right )^{3}=\dfrac{11x^{3}+70x^{2}}{3}$$
$$\Leftrightarrow 3xt^{3}=11x+70$$
$$\Leftrightarrow \dfrac{3t^{3}\left ( 6t+2 \right )}{3t^{2}-7}=\dfrac{11\left ( 6t+2 \right )}{3t^{2}-7}+70$$
$$\Leftrightarrow \left ( t-3 \right )\left ( t+2 \right )\left ( 3t^{2}+4t-13 \right )=0$$
Đối chiếu với điều kiện ta được $t=3$, $t=-2$ và $t=\dfrac{-2-\sqrt{43}}{3}$ từ đó suy ra $x=1$, $x=-2$ và $x=\dfrac{-3-\sqrt{43}}{13+2\sqrt{43}}$.



#649888 Chứng minh $(a+b-ab)(b+c-bc)(c+a-ca)\leqslant 1-abc$

Đã gửi bởi rfiyms on 16-08-2016 - 16:21 trong Bất đẳng thức và cực trị

Câu 1 :Cho a,b,c thỏa mãn : $a+b+c=2$

CMR:$(a+b-ab)(b+c-bc)(c+a-ca)\leqslant 1-abc$

Dấu bằng của bài toán xảy ra khi $a=b=1$ và $c=0$ hoặc các hoán vị chứ không phải xảy ra khi $a=b=c$.

Đặt $a=1-x$, $b=1-y$, $c=1-z$ thì $x+y+z=1$ (vì $a+b+c=2$).

Khi đó:

$$a+b-ab=2-x-y-\left ( 1-x \right )\left ( 1-y \right )=1-xy$$

Tương tự ta được $b+c-bc=1-yz$ và $c+a-ca=1-zx$.

Khi đó bất đẳng thức cần chứng minh trở thành:

$$\left ( 1-xy \right )\left ( 1-yz \right )\left ( 1-zx \right )\leq 1-\left ( 1-x \right )\left ( 1-y \right )\left ( 1-z \right )$$

$$\Leftrightarrow x^{2}y^{2}z^{2}-xyz\left ( x+y+z \right )+x+y+z-1\geq 0$$
$$\Leftrightarrow x^{2}y^{2}z^{2}\geq 1$$
Dấu bằng xảy ra khi $a=b=1$ và $c=0$ hoặc các hoán vị.
 
Cách khác là với bất đẳng thức dạng này cậu có thể xài phương pháp $\text{p, q, r}$. Khi đó từ giả thuyết $a+b+c=2$ bất đẳng thức cần chứng minh trở thành:
$$\left ( ab+bc+ca-abc-1 \right )^{2}\geq 0$$
Bất đẳng thức trên luôn đúng.



#649167 CMR với a,b,c>0 thì: $\sum {\frac{{a^2...

Đã gửi bởi rfiyms on 12-08-2016 - 12:03 trong Bất đẳng thức và cực trị

CMR: với a, b, c > 0 thì:

$\frac{{a^2 }}{{b^2  + c^2 }} + \frac{{b^2 }}{{c^2  + a^2 }} + \frac{{c^2 }}{{a^2  + b^2 }} \ge \frac{a}{{b + c}} + \frac{b}{{c + a}} + \frac{c}{{a + b}}$

Ta có:
\begin{align*} \text{VT}-\frac{3}{2} &=\frac{1}{2}\sum \left ( \frac{a^{2}}{b^{2}+c^{2}}-1 \right ) \\ &=\frac{1}{2}\sum \frac{\left ( a-b \right )\left ( a+b \right )+\left ( a-c \right )\left ( a+c \right )}{b^{2}+c^{2}} \\ &=\frac{1}{2}\sum \left [ \frac{\left ( a-b \right )\left ( a+b \right )}{b^{2}+c^{2}}-\frac{\left ( a-b \right )\left ( a+b \right )}{a^{2}+c^{2}} \right ] \\ &=\frac{1}{2}\left ( a-b \right )\left ( a+b \right )\frac{\left ( a-b \right )\left ( a+b \right )}{\left ( b^{2}+c^{2} \right )\left ( a^{2}+c^{2} \right )} \\ &=\frac{1}{2}\sum \frac{\left ( a-b \right )^{2}\left ( a+b \right )^{2}}{\left ( b^{2}+c^{2} \right )\left ( a^{2}+c^{2} \right )} \end{align*}
Tương tự:
\begin{align*} \text{VP}-\frac{3}{2} &=\frac{1}{2}\sum \frac{a-b+a-c}{b+c} \\ &=\frac{1}{2}\sum \left ( \frac{a-b}{b+c}-\frac{a-b}{a+c} \right ) \\ &=\frac{1}{2}\sum \frac{\left ( a-b \right )^{2}}{\left ( b+c \right )\left ( a+c \right )} \\ \end{align*}
Trừ vế theo vế hai bất đẳng thức trên ta được:
$$\text{VT}-\text{VP}=\frac{1}{2}\sum \left ( a-b \right )^{2}\left [ \frac{\left ( a+b \right )^{2}}{\left ( b^{2}+c^{2} \right )\left ( a^{2}+c^{2} \right )}-\frac{1}{\left ( b+c \right )\left ( a+c \right )} \right ]$$
Theo tiêu chuẩn $2$ của phương pháp $\text{S.O.S}$ bất đẳng thức cuối đúng do đó ta thu được bất đẳng thức cần chứng minh.

 




#648885 Tìm GTNN của $P=\frac{b+2c}{1+a}+\frac...

Đã gửi bởi rfiyms on 10-08-2016 - 11:47 trong Bất đẳng thức và cực trị

Bài 2: Cho $a,b,c\ge 0$ thỏa mãn: $ab+bc+ca=1$. Chứng minh rằng:

$\frac{2a}{a^2+1}+\frac{2b}{b^2+1}+\frac{c^2-1}{c^2+1}\le \frac{3}{2}$.

Có thể sử dụng hệ thức lượng trong tam giác để giải bài này.

Đặt $a=\tan \frac{A}{2}$, $b=\tan \frac{B}{2}$, $c=\tan \frac{C}{2}$ với $A$, $B$, $C\in \left [ 0;\pi  \right )$.
Khi đó giả thuyết trở thành $\tan \frac{A}{2}\tan \frac{B}{2}+\tan \frac{B}{2}\tan \frac{C}{2}+\tan \frac{C}{2}\tan \frac{A}{2}=1$.
Điều này chứng tỏ $A$, $B$, $C$ là ba góc của một tam giác.
Khi đó ta có:
\begin{align*} \frac{2a}{a^{2}+1}+\frac{2b}{b^{2}+1}+\frac{c^{2}-1}{c^{2}+1} &= \frac{2\tan \frac{A}{2}}{\tan ^{2}\frac{A}{2}+1}+\frac{2\tan \frac{B}{2}}{\tan ^{2}\frac{B}{2}+1}+\frac{\tan ^{2}\frac{C}{2}-1}{\tan ^{2}\frac{C}{2}+1} \\ &= \frac{2\tan \frac{A}{2}}{\frac{1}{\cos ^{2}\frac{A}{2}}}+\frac{2\tan \frac{B}{2}}{\frac{1}{\cos ^{2}\frac{B}{2}}}+\frac{\frac{\sin ^{2}\frac{C}{2}-\cos ^{2}\frac{C}{2}}{\cos ^{2}\frac{C}{2}}}{\frac{1}{\cos ^{2}\frac{C}{2}}} \\ &= 2\sin \frac{A}{2}\cos \frac{A}{2}+ 2\sin \frac{B}{2}\cos \frac{B}{2} -\left( \cos ^{2}\frac{C}{2}-\sin ^{2}\frac{C}{2} \right) \\ &= \sin A +\sin B-\cos C \\ & = \sin A+\sin B+\cos \left( A+B \right) \\ &= 2\sin \frac{A+B}{2}\cos \frac{A-B}{2}+1-2\sin ^{2}\frac{A+B}{2} \end{align*}
Mặt khác $\cos \frac{A-B}{2}\leq 1$ và $\sin \frac{A+B}{2}>0$ nên:
\begin{align*} 2\sin \frac{A+B}{2}\cos \frac{A-B}{2}+1-2\sin ^{2}\frac{A+B}{2} &\leq 2\sin \frac{A+B}{2}+1-2\sin ^{2}\frac{A+B}{2} \\ &= \frac{3}{2}-\frac{1}{2}\left( 1-2\sin \frac{A+B}{2} \right)^{2} \leq \frac{3}{2} \end{align*}
Đẳng thức xảy ra khi và chỉ khi $\left\{\begin{matrix} \cos \frac{A-B}{2}=1 &  & \\ \sin \frac{A+B}{2}=\frac{1}{2} & & \\ A+B+C=\pi &  &  \end{matrix}\right.$ giải hệ trên ta được $a=b=2-\sqrt{3}$ và $c=\sqrt{3}$.



#648884 Tìm GTNN của $P=\frac{b+2c}{1+a}+\frac...

Đã gửi bởi rfiyms on 10-08-2016 - 11:31 trong Bất đẳng thức và cực trị

Bài 3: Cho $a,b,c$ không đồng thời bằng $0$ và thỏa mãn: $(a+b+c)^2=2(a^2+b^2+c^2)$. Tìm GTNN,GTLN của:

$P=\frac{a^3+b^3+c^3}{(a+b+c)(ab+bc+ca)}$.

Dự đoán dấu đẳng thức xảy ra khi $a=b$, $c=0$ hoặc $4a=4b=c$ (và các hoán vị).

Ta sẽ chứng minh $\min P=1$. Thật vậy ta có có:

$$\frac{a^{3}+b^{3}+c^{3}}{\left ( a+b+c \right )\left (ab+bc+ca  \right )}\geq 1$$

$$\Leftrightarrow \left (a+b+c  \right )^{3}-3\left ( a+b+c \right )\left ( ab+bc+ca \right )+3abc\geq \left ( a+b+c \right )\left ( ab+bc+ca \right )$$

$$\Leftrightarrow 2\left ( a+b+c \right )\left ( a^{2}+b^{2}+c^{2} \right )+3abc\geq 4\left ( a+b+c \right )\left ( ab+bc+ca \right )$$

$$\Leftrightarrow 4\left ( a+b+c \right )\left ( ab+bc+ca \right )+3abc\geq 4\left ( a+b+c \right )\left ( ab+bc+ca \right )$$

$$\Leftrightarrow 3abc\geq 0$$

Vì $a$, $b$, $c$ không đồng thời bằng $0$ nên đẳng thức xảy ra khi và chỉ khi $a=b$, $c=0$ và các hoán vị.

Ta sẽ chứng minh $\max P=\frac{11}{9}$. Thật vậy, ta có:

$$\frac{a^{3}+b^{3}+c^{3}}{\left ( a+b+c \right )\left ( ab+bc+ca \right )}\leq \frac{11}{9}$$

$$\Leftrightarrow 9\left ( a+b+c \right )^{3}-\frac{27}{4}\left ( a+b+c \right )^{3}+27abc\leq \frac{11}{4}\left ( a+b+c \right )^{3}$$

$$\Leftrightarrow \left ( a+b+c \right )^{3}\geq 54abc$$

Không mất tính tổng quát chuẩn hóa $a+b+c=6$. Đặt $a=x$, $b=y$, $c=4z$. Khi đó $x+y+4z=6$ và $x^{2}+y^{2}+16z^{2}=18$.

Ta sẽ chứng minh $xyz\leq 1$. Ta có:

$$x^{2}-2x+1+y^{2}-2y+1+16\left ( z^{2}-2z+1 \right )\geq 0$$

$$\Leftrightarrow 32z^{2}\leq 32z$$

$$\Leftrightarrow z\leq 1$$

Mặt khác ta lại có:

$$x^{2}+y^{2}+16z^{2}=18$$

$$-2xy+16z^{2}\left ( x+y \right )^{2}-2xy+16z^{2}=18$$

$$\Leftrightarrow \left ( 4z-6 \right )^{2}=18$$

$$\Leftrightarrow 16z^{2}+9=24z+xy$$
$$\Leftrightarrow 16z^{3}+9z=24z^{2}+xyz$$
Ta cần chứng minh $16z^{2}+9z-24z^{2}\leq 1$. Điều này luôn đúng với mọi $z\leq 1$ vì bất đẳng thức tương đương với $\left ( z-1 \right )\left ( 4z-1 \right )^{2}\leq 0$.
Vậy $\min P=1$ đạt được khi $a=b$, $c=0$ và $\max P=\frac{11}{9}$ đạt được khi $4a=4b=c$ và các hoán vị.